Vector space has dimension less than d

Click For Summary
The problem involves demonstrating that a vector space V, spanned by d elements, has a dimension strictly less than d. The key task is to show that the generators of V are not linearly independent, allowing for the removal of one generator while still spanning V. It is clarified that V is not the rational numbers but rather consists of Q-linear combinations of powers of roots of unity. A hint is provided in the question to assist in proving the lack of linear independence. Understanding these concepts is crucial for solving the assignment effectively.
UOAMCBURGER
Messages
31
Reaction score
1

Homework Statement


Problem given to me for an assignment in a math course. Haven't learned about roots of unity at all though. Finding this problem super tricky any help would be appreciated. Screenshot of problem below.
[/B]

53532448_2316637415042677_8609507793054990336_n.png?_nc_cat=110&_nc_ht=scontent.fhlz2-1.png


Homework Equations


Unsure of relevant equations

The Attempt at a Solution


so far i am just trying to understand what the question is asking me to do, i am showing that the vector space V, which is just the rational numbers over the field Q? has dimension strictly less than d, where d is >= 1?[/B]
 

Attachments

  • 53532448_2316637415042677_8609507793054990336_n.png?_nc_cat=110&_nc_ht=scontent.fhlz2-1.png
    53532448_2316637415042677_8609507793054990336_n.png?_nc_cat=110&_nc_ht=scontent.fhlz2-1.png
    11.7 KB · Views: 543
Physics news on Phys.org
The vector space ##V## is spanned by ##d## elements. Therefore, its dimension is at most ##d##. The question asks you to show that the generators are not linearly independent, which means you can remove one of the generators of ##V## such that the remaining generators still span ##V##. It will then follow that the soace has dimension ##d-1##.

The question gives a hint how to show that the generators are not linearly independent.

Also, the space is not the rational numbers.

It are ##\mathbb{Q}##-linear combinations of powers of roots of unity.
 
Question: A clock's minute hand has length 4 and its hour hand has length 3. What is the distance between the tips at the moment when it is increasing most rapidly?(Putnam Exam Question) Answer: Making assumption that both the hands moves at constant angular velocities, the answer is ## \sqrt{7} .## But don't you think this assumption is somewhat doubtful and wrong?

Similar threads

  • · Replies 15 ·
Replies
15
Views
3K
Replies
15
Views
2K
  • · Replies 17 ·
Replies
17
Views
4K
  • · Replies 3 ·
Replies
3
Views
2K
  • · Replies 3 ·
Replies
3
Views
2K
  • · Replies 8 ·
Replies
8
Views
2K
Replies
4
Views
2K
  • · Replies 2 ·
Replies
2
Views
2K
  • · Replies 2 ·
Replies
2
Views
2K
  • · Replies 2 ·
Replies
2
Views
4K